PT91.S4.Q13,14 and Q22: Could a Sage help! Still confused!

Ashley2018-1Ashley2018-1 Live Member
edited November 2021 in Reading Comprehension 2249 karma

If anyone could give a breakdown for these two questions, it would help a lot! For #13, I was able to narrow it down to A and B but don't understand what makes B correct and A incorrect and I'm at a complete loss for Q22.

For 13, I picked A because the passage presented markets as this self correcting thing. It showed how inside information seemed to just spread amongst investors and so I thought the situation in the question stem matched that as well.

As for 14, I thought thermometer matched because the markets according to passage B just gauges majority opinion of investors but why does thermostat match passage A? And what is the difference between a typewriter and a word processor?

For 22, I picked A thinking if it were difficult for international agreements to be put in place to protect endangered species, wouldn't that mean more species would bring about more endangered ones? E just doesn't make sense as an answer to me but it's correct so...

Comments

  • nnnnnnzzzznnnnnnzzzz Member
    177 karma

    For 13, B is supported by passage A that markets help people share information quickly and accurately. A is more like a common assumption?

    For 22, the task is to find a strengthening answer that supports an increase in number of species being associated with an increase in those that need protection. We know that splitters support increasing number of species, and E says that the proponent of the splitter theory are actually pushing for reclassification of species that they know are endangered.

  • Ashley2018-1Ashley2018-1 Live Member
    2249 karma

    @nnnnnnzzzz said:
    For 13, B is supported by passage A that markets help people share information quickly and accurately. A is more like a common assumption?

    For 22, the task is to find a strengthening answer that supports an increase in number of species being associated with an increase in those that need protection. We know that splitters support increasing number of species, and E says that the proponent of the splitter theory are actually pushing for reclassification of species that they know are endangered.

    What do you mean by common assumption? And I know what E is saying my problem is how does that even relate to what we are being asked to do?

  • nnnnnnzzzznnnnnnzzzz Member
    177 karma

    @Ashley2018 said:

    @nnnnnnzzzz said:
    For 13, B is supported by passage A that markets help people share information quickly and accurately. A is more like a common assumption?

    For 22, the task is to find a strengthening answer that supports an increase in number of species being associated with an increase in those that need protection. We know that splitters support increasing number of species, and E says that the proponent of the splitter theory are actually pushing for reclassification of species that they know are endangered.

    What do you mean by common assumption? And I know what E is saying my problem is how does that even relate to what we are being asked to do?

    Oh it may be wrong for me to call it a common assumption, may be something that could be most strongly supported by the question stem is better. It doesn't mean anything besides it's an inference (not a MBT kinda one) after reading the question stem and without taking into consideration of the info discussed in passage A.

    About 22, um because if splitter people actively focus on a subset of species that are endangered, it will make the highlighted sentence more likely don't you think? If they focus on those who are not endangered, doing so may not increase species that need protection.

  • Ashley2018-1Ashley2018-1 Live Member
    edited November 2021 2249 karma

    @nnnnnnzzzz said:

    @Ashley2018 said:

    @nnnnnnzzzz said:
    For 13, B is supported by passage A that markets help people share information quickly and accurately. A is more like a common assumption?

    For 22, the task is to find a strengthening answer that supports an increase in number of species being associated with an increase in those that need protection. We know that splitters support increasing number of species, and E says that the proponent of the splitter theory are actually pushing for reclassification of species that they know are endangered.

    What do you mean by common assumption? And I know what E is saying my problem is how does that even relate to what we are being asked to do?

    Oh it may be wrong for me to call it a common assumption, may be something that could be most strongly supported by the question stem is better. It doesn't mean anything besides it's an inference (not a MBT kinda one) after reading the question stem and without taking into consideration of the info discussed in passage A.

    About 22, um because if splitter people actively focus on a subset of species that are endangered, it will make the highlighted sentence more likely don't you think? If they focus on those who are not endangered, doing so may not increase species that need protection.

    Eh I don’t really understand your explanation for A at all. Did you also get the question wrong? As for E, I disagree. I thought E was completely irrelevant and took the answer out.

  • nnnnnnzzzznnnnnnzzzz Member
    177 karma

    @Ashley2018 said:

    @nnnnnnzzzz said:

    @Ashley2018 said:

    @nnnnnnzzzz said:
    For 13, B is supported by passage A that markets help people share information quickly and accurately. A is more like a common assumption?

    For 22, the task is to find a strengthening answer that supports an increase in number of species being associated with an increase in those that need protection. We know that splitters support increasing number of species, and E says that the proponent of the splitter theory are actually pushing for reclassification of species that they know are endangered.

    What do you mean by common assumption? And I know what E is saying my problem is how does that even relate to what we are being asked to do?

    Oh it may be wrong for me to call it a common assumption, may be something that could be most strongly supported by the question stem is better. It doesn't mean anything besides it's an inference (not a MBT kinda one) after reading the question stem and without taking into consideration of the info discussed in passage A.

    About 22, um because if splitter people actively focus on a subset of species that are endangered, it will make the highlighted sentence more likely don't you think? If they focus on those who are not endangered, doing so may not increase species that need protection.

    Eh I don’t really understand your explanation for A at all. Did you also get the question wrong? As for E, I disagree. I thought E was completely irrelevant and took the answer out.

    I got both of them right :D If it's too hard you can try process of elimination.

    You can ask yourself if A supports any passage. To me it doesn't seem to support passage A at all.

    For 22, I am not sure how E is irrelevant if you truly understand what it said like you said though.

  • Ashley2018-1Ashley2018-1 Live Member
    2249 karma

    @nnnnnnzzzz said:

    @Ashley2018 said:

    @nnnnnnzzzz said:

    @Ashley2018 said:

    @nnnnnnzzzz said:
    For 13, B is supported by passage A that markets help people share information quickly and accurately. A is more like a common assumption?

    For 22, the task is to find a strengthening answer that supports an increase in number of species being associated with an increase in those that need protection. We know that splitters support increasing number of species, and E says that the proponent of the splitter theory are actually pushing for reclassification of species that they know are endangered.

    What do you mean by common assumption? And I know what E is saying my problem is how does that even relate to what we are being asked to do?

    Oh it may be wrong for me to call it a common assumption, may be something that could be most strongly supported by the question stem is better. It doesn't mean anything besides it's an inference (not a MBT kinda one) after reading the question stem and without taking into consideration of the info discussed in passage A.

    About 22, um because if splitter people actively focus on a subset of species that are endangered, it will make the highlighted sentence more likely don't you think? If they focus on those who are not endangered, doing so may not increase species that need protection.

    Eh I don’t really understand your explanation for A at all. Did you also get the question wrong? As for E, I disagree. I thought E was completely irrelevant and took the answer out.

    I got both of them right :D If it's too hard you can try process of elimination.

    You can ask yourself if A supports any passage. To me it doesn't seem to support passage A at all.

    For 22, I am not sure how E is irrelevant if you truly understand what it said like you said though.

    Well if I understood why E was correct I wouldn’t have posted. And I did ask myself whether A was supported by the passage for number 13 and my answer was yes and that’s why I got the question wrong.

  • nnnnnnzzzznnnnnnzzzz Member
    edited November 2021 177 karma

    @Ashley2018 said:

    @nnnnnnzzzz said:

    @Ashley2018 said:

    @nnnnnnzzzz said:

    @Ashley2018 said:

    @nnnnnnzzzz said:
    For 13, B is supported by passage A that markets help people share information quickly and accurately. A is more like a common assumption?

    For 22, the task is to find a strengthening answer that supports an increase in number of species being associated with an increase in those that need protection. We know that splitters support increasing number of species, and E says that the proponent of the splitter theory are actually pushing for reclassification of species that they know are endangered.

    What do you mean by common assumption? And I know what E is saying my problem is how does that even relate to what we are being asked to do?

    Oh it may be wrong for me to call it a common assumption, may be something that could be most strongly supported by the question stem is better. It doesn't mean anything besides it's an inference (not a MBT kinda one) after reading the question stem and without taking into consideration of the info discussed in passage A.

    About 22, um because if splitter people actively focus on a subset of species that are endangered, it will make the highlighted sentence more likely don't you think? If they focus on those who are not endangered, doing so may not increase species that need protection.

    Eh I don’t really understand your explanation for A at all. Did you also get the question wrong? As for E, I disagree. I thought E was completely irrelevant and took the answer out.

    I got both of them right :D If it's too hard you can try process of elimination.

    You can ask yourself if A supports any passage. To me it doesn't seem to support passage A at all.

    For 22, I am not sure how E is irrelevant if you truly understand what it said like you said though.

    Well if I understood why E was correct I wouldn’t have posted. And I did ask myself whether A was supported by the passage for number 13 and my answer was yes and that’s why I got the question wrong.

    Maybe let's try this one method. I think it's helpful for our future as well.

    Try to think like a lawyer and gather evidence or provide reasoning to defend why you think for 13, A is supported by the passage. I said no and present my reasoning: AC A doesn't really touch on the main point of passage A.

    For 22, could you let me know why you think E is irrelevant? If you got the question stem, understood the highlighted sentence, and understood what AC E said, it's really hard to not realize it's a correct answer, much less thinking it's irrelevant.

  • Ashley2018-1Ashley2018-1 Live Member
    edited November 2021 2249 karma

    @nnnnnnzzzz said:

    @Ashley2018 said:

    @nnnnnnzzzz said:

    @Ashley2018 said:

    @nnnnnnzzzz said:

    @Ashley2018 said:

    @nnnnnnzzzz said:
    For 13, B is supported by passage A that markets help people share information quickly and accurately. A is more like a common assumption?

    For 22, the task is to find a strengthening answer that supports an increase in number of species being associated with an increase in those that need protection. We know that splitters support increasing number of species, and E says that the proponent of the splitter theory are actually pushing for reclassification of species that they know are endangered.

    What do you mean by common assumption? And I know what E is saying my problem is how does that even relate to what we are being asked to do?

    Oh it may be wrong for me to call it a common assumption, may be something that could be most strongly supported by the question stem is better. It doesn't mean anything besides it's an inference (not a MBT kinda one) after reading the question stem and without taking into consideration of the info discussed in passage A.

    About 22, um because if splitter people actively focus on a subset of species that are endangered, it will make the highlighted sentence more likely don't you think? If they focus on those who are not endangered, doing so may not increase species that need protection.

    Eh I don’t really understand your explanation for A at all. Did you also get the question wrong? As for E, I disagree. I thought E was completely irrelevant and took the answer out.

    I got both of them right :D If it's too hard you can try process of elimination.

    You can ask yourself if A supports any passage. To me it doesn't seem to support passage A at all.

    For 22, I am not sure how E is irrelevant if you truly understand what it said like you said though.

    Well if I understood why E was correct I wouldn’t have posted. And I did ask myself whether A was supported by the passage for number 13 and my answer was yes and that’s why I got the question wrong.

    Maybe let's try this one method. I think it's helpful for our future as well.

    Try to think like a lawyer and gather evidence or provide reasoning to defend why you think for 13, A is supported by the passage. I said no and present my reasoning: AC A doesn't really touch on the main point of passage A.

    For 22, could you let me know why you think E is irrelevant? If you got the question stem, understood the highlighted sentence, and understood what AC E said, it's really hard to not realize it's a correct answer, much less thinking it's irrelevant.

    lol. It would be perfect if I just understood the question stem and E popped out wouldn’t it? But it didn’t. I didn’t think what the p people thought was relevant. So what if they thought in this certain way? Why would that mean the number of endangered species would increase? And 13 isn’t a main point question. I picked A because I thought the situation in the question stem supported passage A because it showed that the market was self correcting. Passage A talked about how there was insider information that was correct and the information quickly spread which I thought matched what was given in the stem perfectly and so I picked A.

    I think we’re talking in circles. I’ve put in a request for help.

  • nnnnnnzzzznnnnnnzzzz Member
    177 karma

    @Ashley2018 said:

    @nnnnnnzzzz said:

    @Ashley2018 said:

    @nnnnnnzzzz said:

    @Ashley2018 said:

    @nnnnnnzzzz said:

    @Ashley2018 said:

    @nnnnnnzzzz said:
    For 13, B is supported by passage A that markets help people share information quickly and accurately. A is more like a common assumption?

    For 22, the task is to find a strengthening answer that supports an increase in number of species being associated with an increase in those that need protection. We know that splitters support increasing number of species, and E says that the proponent of the splitter theory are actually pushing for reclassification of species that they know are endangered.

    What do you mean by common assumption? And I know what E is saying my problem is how does that even relate to what we are being asked to do?

    Oh it may be wrong for me to call it a common assumption, may be something that could be most strongly supported by the question stem is better. It doesn't mean anything besides it's an inference (not a MBT kinda one) after reading the question stem and without taking into consideration of the info discussed in passage A.

    About 22, um because if splitter people actively focus on a subset of species that are endangered, it will make the highlighted sentence more likely don't you think? If they focus on those who are not endangered, doing so may not increase species that need protection.

    Eh I don’t really understand your explanation for A at all. Did you also get the question wrong? As for E, I disagree. I thought E was completely irrelevant and took the answer out.

    I got both of them right :D If it's too hard you can try process of elimination.

    You can ask yourself if A supports any passage. To me it doesn't seem to support passage A at all.

    For 22, I am not sure how E is irrelevant if you truly understand what it said like you said though.

    Well if I understood why E was correct I wouldn’t have posted. And I did ask myself whether A was supported by the passage for number 13 and my answer was yes and that’s why I got the question wrong.

    Maybe let's try this one method. I think it's helpful for our future as well.

    Try to think like a lawyer and gather evidence or provide reasoning to defend why you think for 13, A is supported by the passage. I said no and present my reasoning: AC A doesn't really touch on the main point of passage A.

    For 22, could you let me know why you think E is irrelevant? If you got the question stem, understood the highlighted sentence, and understood what AC E said, it's really hard to not realize it's a correct answer, much less thinking it's irrelevant.

    lol. It would be perfect if I just understood the question stem and E popped out wouldn’t it? But it didn’t. I didn’t think what the p people thought was relevant. So what if they thought in this certain way? Why would that mean the number of endangered species would increase? And 13 isn’t a main point question. I picked A because I thought the situation in the question stem supported passage A because it showed that the market was self correcting. Passage A talked about how there was insider information that was correct and the information quickly spread which I thought matched what was given in the stem perfectly and so I picked A.

    I think we’re talking in circles. I’ve put in a request for help.

    Um, where in passage A talking about self-correcting? I know that it's not a main point question, but I think it would help you since you kinda didn't get what passage A is talking about.

  • Ashley2018-1Ashley2018-1 Live Member
    edited November 2021 2249 karma

    @nnnnnnzzzz said:

    @Ashley2018 said:

    @nnnnnnzzzz said:

    @Ashley2018 said:

    @nnnnnnzzzz said:

    @Ashley2018 said:

    @nnnnnnzzzz said:

    @Ashley2018 said:

    @nnnnnnzzzz said:
    For 13, B is supported by passage A that markets help people share information quickly and accurately. A is more like a common assumption?

    For 22, the task is to find a strengthening answer that supports an increase in number of species being associated with an increase in those that need protection. We know that splitters support increasing number of species, and E says that the proponent of the splitter theory are actually pushing for reclassification of species that they know are endangered.

    What do you mean by common assumption? And I know what E is saying my problem is how does that even relate to what we are being asked to do?

    Oh it may be wrong for me to call it a common assumption, may be something that could be most strongly supported by the question stem is better. It doesn't mean anything besides it's an inference (not a MBT kinda one) after reading the question stem and without taking into consideration of the info discussed in passage A.

    About 22, um because if splitter people actively focus on a subset of species that are endangered, it will make the highlighted sentence more likely don't you think? If they focus on those who are not endangered, doing so may not increase species that need protection.

    Eh I don’t really understand your explanation for A at all. Did you also get the question wrong? As for E, I disagree. I thought E was completely irrelevant and took the answer out.

    I got both of them right :D If it's too hard you can try process of elimination.

    You can ask yourself if A supports any passage. To me it doesn't seem to support passage A at all.

    For 22, I am not sure how E is irrelevant if you truly understand what it said like you said though.

    Well if I understood why E was correct I wouldn’t have posted. And I did ask myself whether A was supported by the passage for number 13 and my answer was yes and that’s why I got the question wrong.

    Maybe let's try this one method. I think it's helpful for our future as well.

    Try to think like a lawyer and gather evidence or provide reasoning to defend why you think for 13, A is supported by the passage. I said no and present my reasoning: AC A doesn't really touch on the main point of passage A.

    For 22, could you let me know why you think E is irrelevant? If you got the question stem, understood the highlighted sentence, and understood what AC E said, it's really hard to not realize it's a correct answer, much less thinking it's irrelevant.

    lol. It would be perfect if I just understood the question stem and E popped out wouldn’t it? But it didn’t. I didn’t think what the p people thought was relevant. So what if they thought in this certain way? Why would that mean the number of endangered species would increase? And 13 isn’t a main point question. I picked A because I thought the situation in the question stem supported passage A because it showed that the market was self correcting. Passage A talked about how there was insider information that was correct and the information quickly spread which I thought matched what was given in the stem perfectly and so I picked A.

    I think we’re talking in circles. I’ve put in a request for help.

    Um, where in passage A talking about self-correcting? I know that it's not a main point question, but I think it would help you since you kinda didn't get what passage A is talking about.

    When I say self correcting I mean that insider information disseminates.

  • lsat_muselsat_muse Member
    edited November 2021 64 karma

    Question 13:
    The reason you've laid out--the market "self-correcting" and correct information from insiders spreading among investors—matches answer choice B. B says that accurate information spreads quickly throughout the market. And that’s the argument Passage A is making. It would be worth exploring how you matched your reasoning to answer choice A instead of B.

    What answer choice A is saying is that contracts for someone who is initially unpopular CANNOT have a sustained increase in value. For this to be accurate, it needs to pass two tests:
    (1) Is that something Passage A is arguing? Is the author arguing that if someone is initially unpopular they CANNOT/will NEVER have a sustained increase in the value of their contract? No. In fact, the author of passage A doesn’t talk about candidates who are “initially” unpopular at all. Passage A, paragraph 2 just explains how the Iowa Electronic Markets experiment worked. In Passage B, the author talks about a potential weakness in the experiment, which suggests that the “initially unpopular” candidate ended up winning at the last minute. But Passage A doesn’t make this point. So answer choice can’t be right.
    (2) Does the scenario in the question stem show that initially unpopular candidates CANNOT/will NEVER have a sustained increase in value? Tackle this like an LR question. What the scenario shows is that on a single occasion, people tried to manipulate the market and artificially inflate the value of the long-shot candidate.* They succeed temporarily and then the effect disappears. Does that mean that all other attempts will meet a similar fate? No. It’s possible and it could be true that initially unpopular candidates SOMETIMES have a sustained increase in value despite this one example. So answer choice A is inaccurate as well as irrelevant to the argument of Passage A.
    * Also, we have to assume that the long-shot candidate is the initially unpopular candidate.

    Passage A suggests that the market will “self-correct” as you put it but that doesn’t mean that someone who is initially unpopular WILL NEVER have a sustained increase in value. That’s not a matter of “correcting” something in accurate or disseminating insider information. It may be the case that you made an extra assumption there and assumed that “self-correcting” means that the market only sees an increase in value for someone who is always popular (i.e., not “initially unpopular”). But it could be the case that someone who is initially unpopular slowly becomes popular and therefore sees a sustained increase in the value of their contract.

    Question 14:
    Oh man, I really hate this question. I had the same idea as you did for thermometer: it tells me the temperature at a given point in time—just like, according to Passage B, the market “takes the temperature” of majority opinion at given point in time. And a thermostat is used to control or change the temperature—just like, according to Passage A, the market is used to control or change the value of contracts.

    A word processor is like Microsoft Word—a computer program for word processing. So answer choice A was referring to the relationship between a typewriter to a word processor, which may be something like the latter is the modern, techy, evolved version of the former.

    Question 22:
    The author’s final assertion says that increasing the number of recognized species would increase the number of species that need protection. Why does she claim that? Because she assumes that increasing the number of species will also increase the number of endangered species—not just regular species that don’t need protection.

    Let me give an example: we live in a world where there are only 3 species: 1, 2, and 3. You don’t know how many members there are in each group. Only species 3 is endangered and needs protection. The author comes in, looks at this, and says: “Oh, if you increase the total (1+2+3), the total members in 3 are likely to increase also. But why? The increase could be solely in 1, or solely in 2, or solely in 1 and 2. So the author is assuming that the increase in the total number of species will have an increase in the group of endangered species as well.

    If you identify this assumption, you just need to find something to support it and make it “likely” to be true.

    A doesn’t impact the argument. The argument is that increased number of species means increased number of endangered species/species that need protection. Whether or not international agreements are put in place, that doesn’t change the correlation the author has identified in terms of numbers. International agreements to protect endangered species aren’t what determines whether a species is endangered or not. So it really doesn’t impact the numbers and the increase in numbers the author asserts.

    E is difficult to interpret because it requires some simplification and has a conditional statement. It says that “proponents of the phylogenetic species concept” aren’t likely to contest established species classification if no one is endangered.
    - Let’s simplify at the beginning: who are these proponents? They’re splitters. What do they believe? They like to split up known species—which has a net increase in the number of recognized species. This is good because we’re looking for something about numbers increasing.
    - Ok, next up: splitters aren’t likely to contest establish species if no one is endangered. If you have a solid grasp of conditional logic, you can translate this to: /endangered -> /likely to contest. So for regular species, splitters aren’t likely to split up and create more recognized species. Ok, but the author’s assertion is about endangered species so let’s contrapose: likely to contest -> endangered. That means that splitting up is likely to increase the number of endangered species. This is the assumption the author’s argument requires. It shows that an increase in the number of species is likely to be “focused” on the group of endangered species (if you followed my example above, that would be group 3 increase).

  • Ashley2018-1Ashley2018-1 Live Member
    edited November 2021 2249 karma

    @lsat_muse said:
    Question 13:
    The reason you've laid out--the market "self-correcting" and correct information from insiders spreading among investors—matches answer choice B. B says that accurate information spreads quickly throughout the market. And that’s the argument Passage A is making. It would be worth exploring how you matched your reasoning to answer choice A instead of B.

    What answer choice A is saying is that contracts for someone who is initially unpopular CANNOT have a sustained increase in value. For this to be accurate, it needs to pass two tests:
    (1) Is that something Passage A is arguing? Is the author arguing that if someone is initially unpopular they CANNOT/will NEVER have a sustained increase in the value of their contract? No. In fact, the author of passage A doesn’t talk about candidates who are “initially” unpopular at all. Passage A, paragraph 2 just explains how the Iowa Electronic Markets experiment worked. In Passage B, the author talks about a potential weakness in the experiment, which suggests that the “initially unpopular” candidate ended up winning at the last minute. But Passage A doesn’t make this point. So answer choice can’t be right.
    (2) Does the scenario in the question stem show that initially unpopular candidates CANNOT/will NEVER have a sustained increase in value? Tackle this like an LR question. What the scenario shows is that on a single occasion, people tried to manipulate the market and artificially inflate the value of the long-shot candidate.* They succeed temporarily and then the effect disappears. Does that mean that all other attempts will meet a similar fate? No. It’s possible and it could be true that initially unpopular candidates SOMETIMES have a sustained increase in value despite this one example. So answer choice A is inaccurate as well as irrelevant to the argument of Passage A.
    * Also, we have to assume that the long-shot candidate is the initially unpopular candidate.

    Passage A suggests that the market will “self-correct” as you put it but that doesn’t mean that someone who is initially unpopular WILL NEVER have a sustained increase in value. That’s not a matter of “correcting” something in accurate or disseminating insider information. It may be the case that you made an extra assumption there and assumed that “self-correcting” means that the market only sees an increase in value for someone who is always popular (i.e., not “initially unpopular”). But it could be the case that someone who is initially unpopular slowly becomes popular and therefore sees a sustained increase in the value of their contract.

    Question 14:
    Oh man, I really hate this question. I had the same idea as you did for thermometer: it tells me the temperature at a given point in time—just like, according to Passage B, the market “takes the temperature” of majority opinion at given point in time. And a thermostat is used to control or change the temperature—just like, according to Passage A, the market is used to control or change the value of contracts.

    A word processor is like Microsoft Word—a computer program for word processing. So answer choice A was referring to the relationship between a typewriter to a word processor, which may be something like the latter is the modern, techy, evolved version of the former.

    Question 22:
    The author’s final assertion says that increasing the number of recognized species would increase the number of species that need protection. Why does she claim that? Because she assumes that increasing the number of species will also increase the number of endangered species—not just regular species that don’t need protection.

    Let me give an example: we live in a world where there are only 3 species: 1, 2, and 3. You don’t know how many members there are in each group. Only species 3 is endangered and needs protection. The author comes in, looks at this, and says: “Oh, if you increase the total (1+2+3), the total members in 3 are likely to increase also. But why? The increase could be solely in 1, or solely in 2, or solely in 1 and 2. So the author is assuming that the increase in the total number of species will have an increase in the group of endangered species as well.

    If you identify this assumption, you just need to find something to support it and make it “likely” to be true.

    A doesn’t impact the argument. The argument is that increased number of species means increased number of endangered species/species that need protection. Whether or not international agreements are put in place, that doesn’t change the correlation the author has identified in terms of numbers. International agreements to protect endangered species aren’t what determines whether a species is endangered or not. So it really doesn’t impact the numbers and the increase in numbers the author asserts.

    E is difficult to interpret because it requires some simplification and has a conditional statement. It says that “proponents of the phylogenetic species concept” aren’t likely to contest established species classification if no one is endangered.
    - Let’s simplify at the beginning: who are these proponents? They’re splitters. What do they believe? They like to split up known species—which has a net increase in the number of recognized species. This is good because we’re looking for something about numbers increasing.
    - Ok, next up: splitters aren’t likely to contest establish species if no one is endangered. If you have a solid grasp of conditional logic, you can translate this to: /endangered -> /likely to contest. So for regular species, splitters aren’t likely to split up and create more recognized species. Ok, but the author’s assertion is about endangered species so let’s contrapose: likely to contest -> endangered. That means that splitting up is likely to increase the number of endangered species. This is the assumption the author’s argument requires. It shows that an increase in the number of species is likely to be “focused” on the group of endangered species (if you followed my example above, that would be group 3 increase).

    Good morning, so for the markets passage, could I say Passage B is responding to Passage A? Because it appears to be written that way. So the thermostat analogy is conveying Passage A’s presentation of the markets as being able to actually predict outcomes? Just how you can actually set a thermostat (change temperature) while Passage B disagrees with that and says markets don’t necessarily reflect or change reality but only reflect majority opinion.

    As for 13, I guess I just talked myself into answer choice B lol. But as for 13, I get that the author of Passage A didn't explicitly make this point, but he did talk about how insider information (the correct info) was able to spread amongst investors despite them not talking to each other so I thought the point about the long-shot candidate was to say markets know best, aka it corrected itself and if the situation in answer choice A were to happen, then we would get the same outcome.

    Oh dear god. 22 is just terrible. E still sounds like complete gibberish. I understand that splitters like to separate species but how does that lead to increased number of endangered species? That conditional statement doesn’t make any sense to me. What does the likely to contest have to do with anything? When I first read the last sentence of the last passage, I didn't think it had ANY support. Is that correct or was there support given in the previous paragraphs?

    Wait...so likely to contest=likely to split? What?

Sign In or Register to comment.